2019 Mock AMC 10B Problems/Problem 1

Revision as of 00:26, 23 September 2019 by Awsomek (talk | contribs)
(diff) ← Older revision | Latest revision (diff) | Newer revision → (diff)

Simply computing we get: \[40-15\] which equals \[\] $\boxed{\bold{D}}$ $\bold{25}$